Critical Care Final

Ace your homework & exams now with Quizwiz!

A client with colorectal cancer was started on 5-fluorouracil (5-FU) and is experiencing fatigue, diarrhea, and mouth ulcers. A relatively new chemotherapeutic agent, oxaliplatin (Eloxatin), has been added to the treatment regimen. What does the nurse tell the client about the diarrhea and mouth ulcers? A) "A combination of chemotherapeutic agents has caused them." B) "GI problems are symptoms of the advanced stage of your disease." C) "You have these as a result of the radiation treatment." D) "5-FU cannot discriminate between your cancer and your healthy cells."

"5-FU cannot discriminate between your cancer and your healthy cells."

The nurse wishes to reduce the incidence of hospital-acquired acute kidney injury. Which question by the nurse to the interdisciplinary health care team will result in reducing client exposure? A) "Should we filter air circulation?" B) "Can we use less radiographic contrast dye? C) "Should we add low-dose dobutamine? D) "Should we decrease IV rates?"

"Can we use less radiographic contrast dye?"

When assessing a client with acute pyelonephritis, which findings does the nurse anticipate will be present? (Select all that apply) A) Bradycardia B) Vomiting C) Chills D) Dysuria E) Oliguria

-Vomiting -Chills -Dysuria

A client is recovering form an esophagogastroduodenoscopy (EGD) and requests something to drink. What action by the nurse is best? A) Remind the client to remain NPO B) Allow the client cool liquids only C) Tell the client to wait 4 hours D) Assess the client's gag reflex

Assess the client's gag reflex

A 43-year-old client with blunt chest trauma from a motor vehicle accident has a sinus tachycardia, is hypotensive, and has developed muffled heart sounds. There are no obvious signs of bleeding. Which of the following does the nurse suspect? A) Heart failure B) Pneumothorax C) Cardiac tamponade D) Myocardial infarction (MI)

Cardiac tamponade

A ICU nurse is caring for a client with a new diagnosis. The physician comes to the bedside to explain the new diagnosis to the client in more detail. During their conversation, the physician states that the disorder often leads to death without appropriate Renal Replacement Therapy (AKA dialysis). Which condition is the client most likely diagnosed with? A) Acute kidney injury B) Pyelonephritis C) Chronic renal failure D) Peritonitis

Chronic renal failure

A patient taking digoxin (Lanoxin) 0.25 mg and furosemide (Lasix) 40 mg. The patient tells the nurse, "There are yellow halos around the lights." Which action will the nurse take? A) Administer potassium B) Withhold the furosemide C) Evaluate digoxin level D) Document the findings and reassess in 1 hour

Evaluate digoxin levels

Which of the following is not a complication of chronic GERD? A) Esophageal cancer B) Gastric volvulus C) Barrett's esophagus D) Esophageal strictures

Gastric volvulus

A client is brought to the emergency room in an unresponsive state, and diagnosis of hyperglycemic hyperosmolar nonketoic syndrome is made. The client's SaO2 is 99% on RA. The nurse would immediately prepare to initiate which of the following anticipated physician's orders? A) Endotracheal intubation B) Intravenous infusion of normal saline C) Intravenous infusion of sodium bicarbonate D) 100 units of NPH insulin

Intravenous infusion of normal saline

When caring for a client with uremia, the nurse assesses for which symptom? A) Flank pain B) Cyanosis of the skin C) Nausea and vomiting D) Insomnia

Nausea and vomiting

The nurse reviews the arterial blood gas (ABG) values for the client below: ABG: pH, 7.36 PaCO2, 35 HCO3, 26 PaO2, 62 The nurse is most concerned with which value?

PaO2

A client is admitted to the cardiac ICU with a potassium level of 7.6 mmol/L. What change in their ECG rhythm strip will the nurse likely see? A) Peaked T waves B) Absence of P waves C) Widened QRS complex D) Atrial Fibrillation

Peaked T waves

A client with heart failure is taking furosemide (Lasix). Which finding concerns the nurse with this new prescription? A) Serum potassium level of 2.8 mEq/L B) Serum magnesium level of 1.9 mEq/L C) Serum creatinine of 1.0 mg/dL D) Serum sodium level of 135 mEq/L

Serum potassium level of 2.8 mEq/L

Which sensory-perceptual deficit is associated with a left-brain stroke? A) Difficulty judging position and distance B) Slow and possibly fearful performances of tasks C) Overestimation of physical abilities D) Impulsivity and impatience at performing tasks

Slow and possibly fearful performance of tasks

A client who is receiving a blood transfusion suddenly exclaims to the nurse, "I don't feel right!" What does the nurse do next? A) Obtain vital signs and continue to monitor B) Slow the infusion rate of the transfusion C) Stop the transfusion D) Call the Rapid Response Team

Stop the transfusion

The nurse is assessing a client who is receiving mechanical ventilation with positive end-expiratory pressure. Which findings would cause the nurse to suspect a left-sided tension pneumothorax? A) The chest caves in on inspiration and "puffs out" on expiration B) The trachea is deviated to the right side and cyanosis is present C) The left lung field is dull to percussion with crackles present on auscultation D) The client has bloody sputum and wheezes

The trachea is deviated to the right side and cyanosis is present

A nurse witnesses a client begin to experience a tonic-clonic seizures and loss of consciousness. Which action should the nurse take? A) Administer IV push diazepam B) Prepare to intubate the client C) Turn the client's head to the side D) Start fluids via a large-bore catheter

Turn the client's head to the side

What information will the nurse teach the patient who is considering stopping the antiepileptic drug phenytoin? A) You may become confused and delirious B) You may have an acute withdrawal C) You will have severe hypotension D) You may go into status epilepticus

You may go into status epilepticus

An adult patient is brought to the emergency department for treatment of an asthma exacerbation. The patient uses inhaled albuterol as needed to control wheezing. The nurse notes expiratory wheezing, tremors, and restlessness, and a heart rate of 120 beats per minute. The nurse suspects that the patient has A) taken a beta-adrenergic blocker B) not been using albuterol C) taken a monoamine oxidase (MAO) inhibitor D) over-used the albuterol

over-used the albuterol

A patient presents to the ED and is diagnosed with an acute MI. The patient's spouse asks what type of damage has been caused by the "heart attack." What is the appropriate nursing response? A) "The medication will dilate the blood vessels and any damage will be corrected." B) "A heart attack evolves over several hours. We won't know the extent of the damage immediately. C) "The pain is controlled, so there is no damage." D) "It will take years to know the extent of the damage to the heart muscle."

"A heart attack evolves over several hours. We won't know the extent of the damage immediately.

A nurse is teaching a client about compensatory mechanisms that occur during heart failure. She educates the client on the effects of angiotensin on blood pressure. The nurse knows that teaching has been effective when the client states that angiotensin: A) "Makes me urinate more." B) "Constricts my blood vessels." C) "It decreases my heart rate." D) "Dilates my blood vessels

"Constricts my blood vessels."

A nurse cares for a client who is prescribed lactulose (Heptalac). The client states, "I do not want to take this medication because it causes diarrhea." How should the nurse respond? A) "Do not take any more of the medication until your stools firm up." B) "Diarrhea is expected; that's how your body gets ride of ammonia." C) "We will need to send a stool specimen to the laboratory." D) "You may take Kaopectate liquid daily for loose stools."

"Diarrhea is expected; that's how your body gets rid of ammonia."

An intensive care nurse discusses withdrawal of care with a client's family. The family expresses concerns related to discontinuation of therapy. How should the nurse respond? A) "You will need to talk to the provider because I am not legally allowed to participate in the withdrawal of life support." B) "There is no need to worry. Most religious organizations support the client's decision to stop medical treatment." C) "I realize this is a difficult decision. Discontinuation of therapy will allow the client to die a natural death." D) "I understand your concerns, but in this state, discontinuation of care is not a form of active euthanasia."

"I realize this is a difficult decision. Discontinuation of therapy will allow the client to die a natural death."

Which statement indicates to the nurse that the patient understands sublingual nitroglycerin medication instructions? A) "I should sit or lie down after I take a nitroglycerin tablet to prevent dizziness." B) "I will fill my prescription when I start having chest pain." C) "I will take up to five doses every 3 minutes for chest pain." D) "I can chew the tablet for the quickest effect."

"I should sit or lie down after a nitroglycerin tablet to prevent dizziness."

After teaching a client who has an implantable cardioverter-defibrillator (ICD), a nurse assesses the clients understanding. Which statement by the client indicates a correct understanding of the teaching? A) "Now I can discontinue my antidysrhythmic medication." B) "I should participate in a strenuous exercise program." C) "I should wear a snug-fitting shirt over the ICD." D) "I will avoid sources of strong-electromagnetic fields."

"I will avoid sources of strong electromagnetic fields."

After teaching a client with nephrotic syndrome and a normal glomerular filtration, the nurses assesses the client's understanding. Which statement made by the client indicates a correct understanding of the nutritional therapy for this conditions? A) "I must decrease my intake of fat." B) "I will decrease my intake of protein." C) "A decreased intake of carbohydrates will be required." D) "An increased intake of vitamin C is necessary."

"I will decrease my intake of protein."

A nurse is caring for a dying client. The client's spouse states, "I think he is choking to death." The nurse attempted administering anticholinergics which did not resolve the patient's symptoms. What step should the nurse take next? A) "I can ask the respiratory therapist to suction secretions out through his nose." B) "Do not worry. The choking sound is normal during the dying process." C) "I will administer more morphine to keep your husband comfortable." D) "I will have another nurse assist me to turn your husband on his side."

"I will have another nurse assist me to turn your husband on his side."

After teaching a client who is being discharged home after mitral valve replacement surgery, the nurse assesses the client's understanding of his recovery. Which client statement indicates a need for additional teaching? A) "I'll be able to carry heavy loads of after 6 months of rest." B) "I must use an electric razor instead of a straight razor to shave." C) "I will have my teeth cleaned by my dentist in 2 weeks." D) "I must avoid eating foods high in vitamin K, like spinach."

"I will have my teeth cleaned by my dentist in 2 weeks."

Your client is experiencing an evolving myocardial infarction and is being evaluated for thrombolytic therapy. Which statement made by the client would constitute a possible contraindication for this treatment? A) "I've been taking blood pressure medicine for years." B) "I've been having chest pain for 2 hours." C) "I feel really nauseated right now." D) "I'm still taking medicine for my stomach ulcers."

"I'm still taking medicine for my stomach ulcers."

A client has a brain abscess and is receiving phenytoin (Dilantin). The spouse questions the use of the drug, saying the client does not have a seizure disorder. What response by the nurse is best? A) "Preventing febrile seizures with an abscess is important." B) "Seizures always occur in clients with brain abscesses." C) "This drug is used to sedate the client with an abscess." D) "Increased pressure from the abscess can cause seizures."

"Increased pressure from the abscess can cause seizures."

After hiatal hernia repair surgery, a client is on IV pantoprazole (Protonix). The client asks the nurse why this medication is given since there is no history of ulcers. What response by the nurse is best? A) "This operation often causes ulcers." B) "Bacteria can often cause ulcers." C) "The medication keeps your blood pH low." D) "It prevents stress-related ulcers."

"It prevents stress-related ulcers."

A nurse cares for a client with hepatic portal-systemic encephalopathy (PSE). The client is thin and cachectic in appearance, and the family expresses distress that the client is receiving little dietary protein. How should the nurse respond? A) "Less protein in the diet will help prevent confusion associated with liver failure." B) "A low-protein diet will help the liver rest and will restore liver function." C) "Increasing dietary protein will help the client gain weight and muscle mass." D) "Low dietary protein is needed to prevent fluid from leaking into the abdomen."

"Less protein in the diet will help prevent confusion associated with liver failure."

A client has just undergone arterial revascularization. Which statement by the client indicates a need for further teaching related to postoperative care? A) "I should report a fever or any drainage." B) "Throbbing pain with warmness, redness, and swelling are expected." C) "My leg might turn very white after the surgery." D) "I should be concerned if my foot turns blue."

"My leg might turn very white after the surgery."

A client who is scheduled to undergo radiation for prostate cancer is admitted to the hospital by the nurse. Which statement by the client is most important to communicate to the health care provider? A) "I am very tired all of the time." B) "My legs are numb and weak." C) "I am incontinent when I cough." D) "I am allergic to iodine."

"My legs are numb and weak."

The nurse has placed a nasogastric (NG) tube in a client with upper gastrointestinal (GI) bleeding to administer gastric lavage. The client asks the nurse about the purpose of the NG tube for the procedure. What is the nurse's best response? A) "Medication goes down the tube to help clean out your stomach." B) "We'll start feeding you through it once your stomach is cleaned out." C) "The provider requested the tube to be placed just in case it was needed." D) "Saline goes down the tube to help clean out your stomach."

"Saline goes down the tube to help clean out your stomach."

The nursing instructor asks the student nurse to explain a type IV hypersensitivity reaction. Which statement by the student best describes type IV hypersensitivity? A) " The reaction of sensitized T cells with antigen and release of lymphokines activate macrophages and induce inflammation." B) "It results in release of mediators, especially histamine, because of the reaction of immunoglobulin E (IgE) antibody on mast cells." C) "An immune complex of antigen and antibodies is formed and deposited in walls of blood vessels." D) "It is a reaction of immunoglobulin G (IgG) with the host cell membrane or antigen."

"The reaction of sensitized T cells with antigen and release of lymphokines activate macrophages and induce inflammation."

A client present with dark urine, fatigue, and generalized pruritus. Lab results reveal elevated serum bilirubin and increased bile salts. The diagnosis of biliary cirrhosis is made. The client asks what is happening in his body. What is the nurse's best response? A) "Your weakened heart muscle has limited blood flow to the liver." B) "The alcohol you have consumed has caused small nodules to form in your liver." C) "Your liver is releasing toxins, which are poisonous to your circulation." D) "There is an obstruction of the bile ducts causing biliary inflammation."

"There is an obstruction of the bile ducts causing biliary inflammation."

A client with HIV/AIDS asks the nurse why gabapentin (Neurontin) is part of the drug regimen when the client does not have a history of seizures. What response by the nurse is best? A) "This drug helps treat the pain from nerve irritation." B) "You are at risk for seizures due to fungal infections." C)"I have no idea why you should be taking this drug." D)"Gabapentin can be used as an antidepressant too."

"This drug helps treat the pain from nerve irritation."

An HIV-negative client who has an HIV-positive partner asks the nurse about receiving Truvada (emtricitabine and tenofovir). What information is most important to teach the client about this drug" A) "Truvada does not reduce the need for safe sex practices." B) "Truvada reduces the number of HIV tests you will need." C) "This drug has been taken off the market due to increases in cancer." D) "This drug is only used for postexposure prophylaxis."

"Truvada does not reduce the need for safe sex practices."

A nurse teaches a client who is recovering form a nephrectomy secondary to kidney trauma. Which statement should the nurse include in this client's teaching? A) "You need to avoid participating in contact sports like football." B) "Medication will be prescribed to control your high blood pressure." C) "Your therapy will include hemodialysis while you recover." D) "Since you only have one kidney, a salt and fluid restriction is required."

"You need to avoid participating in contact sports like football."

The nurse is assessing a client who is experiencing substernal chest pain? Which report of symptoms supports a diagnosis of stable angina pectoris rather than a myocardial infarction? (Select all that apply) A) "The pain started when I was eating breakfast and continued all morning." B) "I feel a lot better when I sit down and rest for a little while." C) "I took two nitroglycerin tablets and the pain went away." D) "The only thing that helps with the pain is Morphine." E) "The pain lasts less than five minutes."

- "I feel a lot better when I sit down and rest for a little while." - "I took two nitroglycerin tablets and the pain went away." - "The pain lasts less than five minutes."

A client is scheduled to start external beam radiation therapy (EBRT) for her endometrial cancer. Which teaching by the nurse is accurate? (Select all that apply) A) "You will need to be hospitalized during this therapy." B) "It is okay to clean off skin markings. The will be reapplied before each treatment." C) "Wash the irradiated area gently each day with water and mild soap." D) "Your skin needs to be inspected daily for any breakdown." E) "It is not wise to stay out in the sun for long periods of time."

-"Wash the irradiated area gently each day with water and mild soap." -"Your skin needs to be inspected daily for any breakdown." -"It is not wise to stay out in the sun for long periods of time."

A nurse cares for a client with burn injuries during the resuscitation phase. Which actions are priorities during this phase? (Select all that apply) A) Administer analgesics B) Initiate physical therapy C) Provide fluid replacement D) Prevent wound infections E) Decrease core temperature

-Administer analgesics -Provide fluid replacement -Prevent wound infections

The nurse is caring for a client with suspected severe sepsis. What does the nurse prepare to do within 3 hours of the client being identified as being at risk? (Select all that apply) A) Administer antibiotics B) Infuse vasopressors C) Draw serum lactate levels D) Obtain blood cultures E) Measure central venous pressure

-Administer antibiotics -Draw serum lactate levels -Obtain blood cultures

A client is being admitted with suspected tuberculosis (TB). What actions by the nurse are best? (Select all that apply) A) Other than wearing gloves, no special actions are needed B) Admit the client to a negative-airflow room C) Maintain a distance of 6 feet from the client at all times D) Wash hands with chlorhexidine after providing care E) Order specialized masks/respiratory for caregiving

-Admit the client to a negative-airflow room -Order specialized masks/respirators for caregiving

A client receiving chemotherapy has a white blood cell count of 1000/mm3 during the nadir of their bone marrow activity. What actions by the nurse are most appropriate? (Select all that apply) A) Check the patient's temperature once every shift. B) Assess all mucous membranes every 4 to 8 hours C) Monitor and record vital signs every 4 hours D) Do NOT administer enemas E) Monitor the venous access device appearance every hour while chemotherapy is being administered

-Assess all mucous membranes every 4 to 8 hours -Monitor and record vital signs every 4 hours -Monitor the venous access device appearance every hour while chemotherapy is being administered

The nurse is caring for client who has developed bradycardia. Which of the following statements are true regarding bradycardia? (Select all that apply) A) Bearing down for a bowel movement may cause bradycardia B) Symptomatic bradycardia may be treated with atropine C) No treatment is necessary for mild, asymptomatic bradycardia D) Bradycardia appears on ECG as an irregular, slow rate with a wide QRS complex F) The second line treatment option for symptomatic bradycardia is immediate defibrillation

-Bearing down for a bowel movement may cause bradycardia -Symptomatic bradycardia may be treated with atropine -No treatment is necessary for mild, asymptomatic bradycardia

Which clients are not candidates for tissue plasminogen activator (tPA) for the treatment of stroke? A) Blood pressure of 200/110 B) Signs of aphasia and ataxia C) Hemorrhagic stroke 3 years ago D) Ischemic stroke last year E) Negative CT scan F) Received Heparin 24 hours ago

-Blood pressure of 200/110 -Hemorrhagic stroke 3 years ago -Received Heparin 24 hours ago

A nurse admits an older adult client to the hospital. Which criterion should the nurse use to determine if the client can make his own medical decisions? (Select all that apply) A) Can communicate his treatment preferences B) Is oriented enough to understand information provided C) Can evaluate and deliberate information D) Is able to read and write at an eighth-grade level E) Has completed an advance directive

-Can communicate his treatment preferences -Is oriented enough to understand information provided -Can evaluate and deliberate information

The nurse suspects that a client has developed an acute arterial occlusion of the right lower extremity based on which signs/symptoms? ( Select all that apply) A) Cold right foot B) Numbness and tingling of right foot C) Bounding right pedal pulses D) Hypertension E) Tachycarida F) Mottling of right foot and lower leg

-Cold right foot -Numbness and tingling of right foot -Mottling of right foot and lower leg

An emergency room nurse assesses a client with potential liver trauma. Which clinical manifestation should alert the nurse to internal bleeding and hypovolemic shock? (Select all that apply) A) Confusion B) Tachycardia C) Flushed, warm skin D) Bradypnea E) Hypertension

-Confusion -Tachycardia

A nurse teaches a client's family members about signs and symptoms of approaching death. Which manifestations should the nurse include in this teaching? (Select all that apply) A) Congestion and gurgling B) Warm and flushed extremities C) Decreased appetite D) Difficulty sleeping E) Irregular and slowed breathing

-Congestion and gurgling -Decreased appetite -Irregular and slowed breathing

A nurse assesses a client with hypothyroidism who is admitted with acute appendicitis. The nurse notes that the client's level of consciousness has decreased. Which actions should the nurse take? (Select all that apply) A) Cover the client with warm blankets B) Infuse intravenous fluids C) Monitor blood pressure every shift D) Administer oral glucose as prescribed E) Maintain a patent airway

-Cover the client with warm blankets -Infuse intravenous fluids -Maintain a patent airway

A client who had a myocardial infarction asks why he is receiving morphine. Which benefits of morphine should the nurse explain to this client? (Select all that apply) A) Diminished anxiety B) Decreased myocardial oxygen demand C) Vasoconstriction of peripheral vessels D) Increased urinary output E) Pain relief

-Diminished anxiety -Decreased myocardial oxygen demand -Pain relief

The nurse caring for mechanically ventilated clients use best practices to prevent ventilator-associated pneumonia. What actions are included in this practice? (Select all that apply) A) Suctioning the client on a regular schedule B) Elevating the head of the bed C) Providing oral care per protocol D) Adherence to proper hand hygeine E) Administering anti-ulcer medication

-Elevating the head of the bed -Providing oral care per protocol -Adherence to proper hand hygiene -Administering anti-ulcer medication

When caring for a client with portal hypertension, the nurse assesses for which potential complications? (Select all that apply) A) Esophageal varices B) Fever C) Ascites D) Hematuria E) Hemorrhoids

-Esophageal varices -Ascites -Hemorrhoids

The nurse is concerned that a client who had myocardial infarction (MI) has developed cardiogenic shock. Which findings indicate this type of shock? (Select all that apply) A) Faint heart sounds B) Respiratory rate of 12 breaths/min C) Temperature of 100.4 F D) Cool, diaphoretic skin E) Anxiety and restlessness F) Bradycardia

-Faint hear sounds -Cool, diaphoretic skin -Anxiety and restlessness

A nurse assess a client with a brain tumor. Which newly identified assessment findings should alert the nurse to urgently communicate with the health care provider? ( Select all that apply) A) Uninhibited speech B) Glasgow Coma Scale score of 8 C) Decerebrate posturing D) Reactive pupils E) Diminished cognition

-Glasgow Coma Scale score of 8 -Decerebrate posturing -Diminished cognition

A nurse assesses a client who experienced a spinal cord injury at the T5 level 12 hours ago. Which manifestations should the nurse correlate with neurogenic (spinal) shock? (Select all that apply) A) Heart rate of 34 beats/min B) Decreased level of consciousness C) Increased oxygen saturation D) Blood pressure of 185/65 mm Hg E) Loss of knee jerk reflex

-Heart rate of 34 beats/min -Decreased level of consciousness -Loss of knee jerk reflex

The nurse administers amiodarone (Cordarone) to a client with ventricular tachycardia. Which monitoring by the nurse is necessary with this drug? (Select all that apply) A) Heart rhythm B) Heart rate C) Respiratory rate D) Urine output E) QT interval

-Heart rhythm -Heart rate -QT interval

The student nurse studying shock understands that the common manifestations of this condition are directly related to which problems? (Select all that apply) A) Hypotension B) Dyspnea C) Anaerobic metabolism D) Increased perfusion E) Hyperglycemia

-Hypotension -Anaerobic metabolism

The nurse is caring for the client with increased intracranial pressure. The nurse would notice which of the following trends in vital signs if the intracranial pressure is rising: (Select all that apply) A) Increasing blood pressure B) Decreasing respirations C) Increasing heart rate D) Decreasing blood pressure E) Increasing respirations F) Decreasing heart rate

-Increasing blood pressure -Decreasing respirations -Decreasing heart rate

Which finding are AIDS-defining characteristics? (Select all that apply) A) Infection with Pneumocystis jiroveci B) Presence of HIV wasting syndrome C) Positive enzyme-linked immunosorbent assay (ELISA) test for human immune deficiency virus (HIV) D) CD4+ cell count less than 200/mm3 or less than 14% E) Taking antiretroviral medications

-Infection with Pneumocystis jiroveci -Presence of HIV wasting syndrome -CD4+ cell count less than 200/mm3 or less than 14%

A nurse plans care for a client with epilepsy who is admitted to the hospital. Which interventions should the nurse include in this client's plan of care? (Select all that apply) A) Keep padded bed rails up at all times B) Have suction equipment at the bedside C) Maintain the client on strict bedrest D) Place a padded tongue blade at the bedside E) Ensure that the client has IV access F) Permit only clear oral fluids

-Keep padded bed rails up at all times -Have suction equipment at the bedside -Ensure that the client has IV access

A nurse prepares a client for a percutaneous kidney biopsy. Which actions should the nurse take prior to this procedure? (Select all that apply) A) Keep the client NPO B) Obtain platelet lab results C) Maintain strict bedrest in a supine position D) Assess for blood in the client's urine E) Administer antihypertensive medications

-Keep the client NPO -Obtain platelet lab results -Administer antihypertensive medications

A client is hospitalized in the oliguric phase of acute kidney injury (AKI) and is receiving tube feedings. The nurse is teaching the client's spouse about the kidney-specific formulation for the enteral solution compared to standard formulas. What component should be discussed in the teaching plan? ( Select all that apply) A) Lower protein B) Higher calcium C) Lower potassium D) Higher phosphorus E) Higher calories

-Lower protein -Lower potassium -Higher calories

A student nurse is learning about human immune deficiency virus (HIV) infection. Which statements about HIV infection are correct? (Select all that apply) A) People with stage 1 HIV are not infectious to others B) Macrophages stop functioning properly C) Opportunistic infections and cancer are leading causes of death D) Antibodies produced are mostly incomplete and do not function well E) CD4+ cells begin to create new HIV virus particles

-Macrophages stop functioning properly -Opportunistic infections and cancer are leading causes of death -Antibodies produced are mostly incomplete and do not function well -CD+ cells begin to create new HIV virus particles

A nurse is caring for a client on IV infusion of heparin. What actions does this nurse include in the client's plan of care? (Select all that apply) A) Stop the IV for aPTT 1.5 times above the baseline value B) Monitor the daily activated partial thromboplastin time (aPTT) results C) Assess the client for bleeding D) Use an IV pump for the infusion E) Strictly monitor intake and output

-Monitor the daily activated partial thromboplastin time (aPTT) results -Assess the client for bleeding -Use an IV pump for the infusion

A client has recently been released from prison and has just tested positive for tuberculosis (TB). What teaching does the community health nurse want to stress for this client regarding medications? (Select all that apply) A) Not taking the medication could lead to an infection that is difficult to treat or to total drug resistance B)Avoid alcoholic beverages while taking these drugs C) These medications may cause kidney failure D) The medication may cause nausea. The client should take them at bedtime E) The client is generally not contagious after 2 to 3 consecutive weeks of treatment

-Not taking the medication could lead to an infection that is difficult to treat or to total drug resistance -The medications may cause nausea. The client should take them at bedtime

A client was intubated 30 minutes ago for acute respiratory distress syndrome and possible sepsis. The following orders have been given for the client. In what sequences would the nurse perform these orders for this client? 1) Infuse levofloxacin (Levaquin) 500 mg IV 2) Obtain baseline aerobic and anaerobic sputum cultures 3) Teach the client and family methods of communicating 4) Analyze postintubation arterial blood gases (ABGs)

-Obtain baseline aerobic and anaerobic sputum cultures -Infuse levofloxacin (Levaquin) 500 mg IV -Analyze postintubation arterial blood gases (ABGs) -Teach the client and family methods of communicating

A client with an infection has a fever. What actions by the nurse help increase the client's comfort? (Select all that apply) A) Offer cool fluids to the client frequently B) Administer antipyretics around clock C) Change the client's gown and linens when damp D) Provide a fan to help cool the client

-Offer cool fluids to the client frequently -Change the client's gown and linens when damp

A nurse assesses a client who is experiencing an absence seizure. For which clinical manifestations should the nurse assess? (Select all that apply) A) Intermittent rigidity B) Brief jerking of the extremities C) Picking at clothing D) Sudden loss of muscle tone E) Blank stare F) Lip smacking

-Picking at clothing -Blank stare -Lip smacking

A nurse cares for a client with pancreatic cancer who is prescribed implanted radioactive iodine seeds. Which action should the nurse take when caring for this client? (Select all that apply) A) Place the client in a private room B) Initiate Transmission-Based Precautions. C) Bundle care to minimize exposure to the client. D) Dispose of dirty sheets and blankets in a blue linen bag and call Environmental Services to remove the bag from the room. E) Wear a lead apron when providing client care.

-Place the client in a private room -Bundle care to minimize exposure to the client -Wear a lead apron when providing client care

A nurse is assessing a client with left-sided heart failure. For which clinical manifestations should the nurse assess? (Select all that apply) A) Pulmonary crackles B) Cough that worsens at night C) Ascites D) Dependent edema E) Confusion, restlessness

-Pulmonary crackles -Cough that worsens at night -Confusion, restlessness

The nurse cares for a client who is prescribed vasopressin (DDAVP) for diabetes insipidus. Which assessment findings indicate a therapeutic response to this therapy? (Select all that apply) A) Specific gravity is increased B) Urine output is increased C) Blood pressure is increased D) Specific gravity is decreased E) Urine output is decreased F) Blood pressure is decreased

-Specific gravity is increased -Blood pressure is increased -Urine output is decreased

A client has a gastrointestinal hemorrhage and is prescribe two units of packed red blood cells. What actions should the nurse perform prior to hanging the blood? (Select all that apply) A) Teach the client about reaction manifestations B) Take and record a set of vital signs C) Prime the IV tubing with normal saline D) Ask a second nurse to double-check the blood E) Prime the IV tubing the dextrose in water

-Teach the client about reaction manifestations - Take and record a set of vital signs -Prime the IV tubing with normal saline -Ask a second nurse to double-check the blood

The nurse is educating a client about transient ischemic attacks (TIAs). Which of the following statements, if made by the nurse, are correct regarding TIAs? (Select all that applies) A) They serve as a warning sign that an impending stroke may occur B) They do not require medical treatment C) They are caused by a temporary decrease in blood flow to the brain D) They produce signs and symptoms that can last for several weeks to months

-They serve as a warning sign that an impending stroke may occur -They are caused by a temporary decrease in blood flow to the brain

The nursing student is studying hypersensitivity reactions. Which reactions are correctly matched with their hypersensitivity types? (Select all that apply) A) Type IV-Examples include poison ivy and graft rejection B) Type I-Examples include latex allergy and anaphylaxis C) Type IV-Mediated by action of immunoglobulin E (IgE) D) Type II-Examples include scarcoidosis and systemic lupus erythematosus E) Type III-Examples include serum sickness and rheumatoid arthritis

-Type IV-Examples include poison ivy and graft rejection -Type I-Examples include latex allergy and anaphylaxis -Type III-Examples include serum sickness and rheumatoid arthritis

When monitoring a client with suspected syndrome of inappropriate antidiuretic hormone (SIADH), the nurse reviews the client's medical record, which contains the following information. The nurse notifies the health care provider for which signs and symptoms consistent with this syndrome? (Select all that apply) Neuro: Episodes of confusion Cardiac: Pulse 88 and regular Musculoskeletal: Weakness, tremors Na: 115 K: 4.2 Creatinine: 0.8 Medications: ondansetron (Zofran) and cyclophosphamide (Cytoxan) A) Weakness B) Mental status changes C) Azotemia D) Hyponatremia E) Bradycardia

-Weakness -Hyponatremia -Mental status changes

A nurse assesses a client who has encephalitis. Which manifestations should the nurse recognize as signs of increased intracranial pressure (ICP), a complication of encephalitis? (Select all that apply) A) Widened pulse pressure B) Irregular respirations C) Bradycardia D) photophobia E) Headache

-Widened pulse pressure -Irregular respirations -Bradycardia

An emergency room nurse cares for a client admitted with a 50% burn injury at 10:00 this morning. The client weighs 90 kg. Using the Parkland formula, calculate the rate at which the nurse should infuse intravenous fluid resuscitation when started at noon. (Record your answer using the whole number.)

1,500

A nurse uses the rule of nines to assess a client with burn injuries to the entire back region, posterior left arm, and posterior right arm. How should the nurse document the percentage of the client's body that sustained burns? A) 27% B) 18% C) 9% D) 36%

27%

In order for tissue plasminogen activator (tPA) to be most effective in the treatment of stroke, it must be administered? A) 6 hours after the onset of stroke symptoms B) 3 hours before the onset of stroke symptoms C) 12 hours after the onset of stroke symptoms D) 3 hours after the onset of stroke symptoms

3 hours after the onset of stroke symptoms

The nurse is caring for a 50 year old male client who has suffered second and third degree burns to the entire anterior thorax, anterior right arm, and the anterior of the left upper leg. The client weights 80 kg. Using the Parkland Formula for fluid resuscitation, the patient should receive ___________ fluids in the first 24 hours post injury.

8,640

A nurse is taking care of four clients on his unit. Which of the following client's would the nurse know is most likely to be diagnosed with acute rheumatic carditis? A) A client who presents with redness, swelling, and pain in her right leg B) A client with chest pain relieved by rest C) A client with jugular venous distention and muffled heart sounds on ausculatation D) A client who reports having a "sore throat" last week who present with a new onset heart murmur

A client who reports having a "sore throat" last week who presents with a new onset heart murmur

A charge nurse is assigning a staff nurse to care for a client who is prescribe implanted radioactive iodine seeds. Which staff nurses should be excluded when considering who should provide this client's care? A) A staff nurse who is pregnant B) An agency nurse who has come to the unit to help C) A newly hired graduate nurse D) A staff nurse fro the float pool

A staff nurse who is pregnant

Which finding in the first 24 hours after kidney transplantation requires immediate intervention? A) Abrupt decrease in urine output B) Blood-tinged urine C) Incisional pain D) Increase in urine output

Abrupt decrease in urine output

Which client has the least risk of developing an acute kidney injury (AKI) A) Client who underwent contrast dye radiology B) Client recovering from severe gastrointestinal influenza C) Client in the intensive care unit on high dose of antibiotics D) Accident victim recovering from a severe hemorrhage E) Accountant with diabetes F) Football player in preseason practice

Accountant with diabetes

Which client is not at risk for acute kidney injury? (choose 1) A) Football player in preseason practice B) Client who underwent contrast dye radiology C) Accident victim recovering from severe hemorrhage D) Accountant with diabetes E) Client in the intensive care unit on high doses of antibiotics F) Client recovering from gastrointestinal influenza

Accountant with diabetes

Your patient has a blood potassium level of 9.2 mEq/L. What intervention should you anticipate for this patient? A) Intravenous calcium supplementation B) Intravenous potassium supplementation C) Administer Sodium Polystyrene (Kayexalate) D) Parenteral nutrition

Administer Sodium Polystyrene (Kayexalate)

A nurse cares for a client with deep partial thickness burn injuries. Which intervention should the nurse implement to appropriately reduce the client's pain? A) Administer the prescribed intravenous morphine sulfate B) Decrease tactile stimulation near the burn injuries C) Administer prescribed intramuscular ketorolac (Toradol) D) Apply ice to skin around the burn wound for 20 minutes

Administer the prescribed intravenous morphine sulfate

After a stroke, a client has ataxia. What intervention is most appropriate to include on the client's plan of care? A) Encourage double swallowing B) Monitor lung sounds after eating C) Perform post-void residuals D) Ambulate only with a gait belt

Ambulate only with a gait belt

An ICU nurse admits a client with acute onset confusion. During the initial assessment, the client wakes only to sternal rub and appropriately follows commands for short period of time. The client is oriented to self. The nurse notes a positive flapping tremor and palmar erythema. What lab value should the nurse expect to draw determine the client's admitting diagnosis? A) Albumin B) Amylase C) Bilirubin D) Ammonia

Ammonia

A client will receive I.V. midazolam hydrochloride (Versed) during surgery. Which of the following should the nurse determine as a therapeutic effect? A) Amnesia B) Decreased blood pressure C) Paralysis D) Anesthesia

Amnesia

Aside from chemotherapeutic agents, what other medications does the nurse expect to administer to a client with advanced colorectal cancer for relief of symptoms" A) Steroids and analgesics B) Steroids and anti-inflammatory medications C) Analgesics and benzodiazepines D) Analgesics and antiemetics

Analgesics and antiemetics

A client calls the clinic to report exposure to poison ivy and an itchy rash that is not helped with over-the-counter antihistamines. What responses by the nurse is most appropriate? A) There are different antihistamines to try B) Make sure that your medication is not expired C) You should be seen in the clinic right away D) Antihistamines do not help poison ivy

Antihistamines do not help poison ivy

A client is experiencing the classic signs and symptoms of unstable, acute coronary disease. What is the nurse's priority intervention? A) Apply supplemental oxygen B) Administer sublingual nitroglycerin C) Remain with the client in order to decrease anxiety D) Educate the client about the pathophysiology causing his symptoms

Apply supplemental oxygen

A client has died after a long hospital stay. The family was present at the time of the client's death. Which postmortem action does the nurse implement? A) Raises the head of the bed and opens the client's eyes B) Removes dentures and any prosthetics C) Asks the family if they wish to help wash the client D) Asks the family to leave

Asks the family if they wish to help wash the client

A nurse cares for a client who is on cardiac monitor. The monitor displayed the rhythm shown below. (Looks like lots of large hills) Which action should the nurse take first? A) Begin cardiopulmonary resuscitation (CPR) B) Administer an amiodarone bolus followed by a drip C) Cardiovert the client with biphasic defibrillator D) Assess airway, breathing, level of consciousness, and pulse rate

Assess airway, breathing , level o consciousness, and pulse rate

A nurse assesses a client who is recovering from a subtotal thyroidectomy. On the second postoperative day the client states, "I feel numbness and tingling around my mouth." What action should the nurse take? A) Assess for Chvostek's sign B) Offer mouth care C) Loosen the dressing D) Ask the client orientation questions

Assess for Chvostek's sign

A client with human immune deficiency virus (HIV) has a sudden decline in status with a large increase in viral load. What action should the nurse take first? A) Request information about new living quarters or pets B) Assess the client for adherence to the drug regimen C) Ask the client about travel to any foreign countries D) Determine i the client has any new sexual partners

Assess the client for adherence to the drug regimen

The peak pressure alarm is sounding on the ventilator of a client with a recent tracheostomy. what intervention should be done first? A) Ensure that the connecting tubing is not kinked B) Decrease the sensitivity of the alarm C) Suction the client D) Assess the client's respiratory status

Assess the client's respiratory status

A client had recent unprotected sex with an HIV positive partner. He came into the clinic yesterday & had an enzyme-linked immunosorbent assay (ELISA) test drawn for human immune deficiency virus (HIV) antibodies. A nurse is now talking with a client about the results from that test. The test is negative and the client state "Whew! I was really worried about that result." What action by the nurse is most important? A)Express happiness over the test result B) Assess the client's sexual activity and patterns C)Remind the client about safer sex practices D)Tell the client to get retested in 2 weeks

Assess the client's sexual activity and patterns

A nurse cares for a client who is recovering from a closed percutaneous kidney biopsy. The client states, "My pain has suddenly increased from a 3 to a 10 on a scale of 0 to 10." Which action should the nurse take first? A) Reposition the client on the operative side B) Administer the prescribed opioid analgesic C) Assess the pulse rate and blood pressure D) Examine the color of the client's urine

Assess the pulse rate and blood pressure

After a hypophysectomy, a client complains of being thirsty hand having to urinate frequently. The initial nursing action is to take is to: A) Document the complaints B) Increase fluid intake C) Assess urine specific gravity D) Assess for urinary glucose

Assess urine specific gravity

A client has been intubated and placed on a volume-cycled mechanical ventilator. The nurse carefully assesses the client for finding associated with a risk associated with this type of ventilator. What is that risk? A) Hypoventialtion B) Barotrauma C) Hypercapnea D) Respiratory acidosis

Barotrauma

The nurse is caring for a client with impending respiratory failure who refuses intubation and mechanical ventilation. Which method provides an alternative to mechanical ventilation? A) Bi-level positive airway pressure (BiPAP) B) Oropharyngeal airway C) Positive end-expiratory pressure (PEEP) D) Non-rebreather mask with 100% oxygen

Bi-level positive airway pressure (BiPAP)

Which clinical manifestations does the nurse recognize indicates worsening in the condition of a patient in the refractory phase of shock? A) Urine output of 20 mL/hr B) Increasing respiratory rate C) Bleeding, oozing from IV sites D) Warm, flushed skin

Bleeding, oozing from IV sites

For a client with an 8-cm abdominal aneurysm, which problem must be addressed immediately to prevent rupture? A) Anxiety B) Blood pressure 192/102 mm Hg C) Report of constipation D) Heart rate 52 beats/min

Blood pressure 192/102 mm Hg

A client with septic shock has been started on dopamine (Intropin) at 12 mcg/kg/min. Which response indicates a positive outcome? A) Serum creatinine 3.6 mg/dL B) Blood pressure 90/60 mm Hg and mean arterial pressure 70 mm Hg C) Blood glucose 245 mg/dL D) Hourly urine output 10 to 12 mL/hr

Blood pressure 90/60 mm Hg and mean arterial pressure 70 mm Hg

A client in the intensive care unit is started on continuous renal replacement therapy (CRRT). Which finding is the cause of immediate action by the nurse? A) Blood pressure of 76/58 mm Hg B) Sodium level of 138 mEq/L C) Potassium level of 5.5 mEq/L D) Pulse rate of 90 beats/min

Blood pressure of 76/58 mm Hgq

A client is admitted to the hospital with a temperature o 95 F (35 C) and high white blood cell count. Which intervention should the nurse anticipate initiating? A) Cooling baths B) Blood transfusion C) Broad-spectrum antibiotics D) NPO status

Broad-spectrum antibiotics

Epinephrine is used to treat cardiac arrest and status asthmaticus because of which of the following actions? A) Bronchoconstriction and increased heart rate B) Bronchodilation and increased heart rate, contractility, and conduction C) Increased vasodilation and enhanced myocardial contractility D) Increased speed of conduction and gluconeogeneis

Bronchodilation and increased heart rate, contractility, and conduction

A client with hypovolemic shock has these vital signs: temperature 97.9 F; pulse 122 beats/min; blood pressure 86/48 mm Hg; respirations 24 breaths/min; urine output 20 mL for last 2 hours; skin cool and clammy. Which medication order for this client does the nurse question? A) Bumetanide (Bumex) B) Dobutamine (Dobutrex) C) Primacor (Milrinone) D) Dopamine (Intropin)

Bumetanide (Bumex)

The nurse is assessing a client with a diagnosis of pre-renal acute kidney injury (AKI). Which condition would the nurse expect to find in the client's recent history? A) Pyelonephritis B) CHF C) Bladder cancer D) Kidney stones

CHF

A client complaining of severe substernal pain is presently being seen in the emergency department. An acute anterior myocardial infarction is suspected. An elevation of which enzyme would confirm the diagnosis at this time? A) Myoglobin B) CK-MB C) C-reactive protein D) BNP

CK-MB

The nurse is assessing a client with neurologic condition who is reporting difficulty chewing when eating. The nurse suspects that which cranial nerve has been affected? A) CN VII-Facial B) CN IV-Trochlear C) CN VI-Abducens D) CN V-Trigeminal

CN V-Trigeminal

A client is experiencing chest pain at rest. The pain is unresponsive to Nitroglycerine. The client is diagnosed with unstable angina, and the nurse immediately begins to prepare the patient for intervention. What treatment is most appropriate for this client? A) Echocardiogram B) Defibrillation C) Heart transplantation D) Cardiac catheterization

Cardiac catheterization

Which finding alarms the nurse most when caring for a client receiving chemotherapy who has a platelet count of 17,000/mm3? A) Change in mental status B) Weigh gain of 4 pounds in 1 day C) Increasing shortness of breath D) Diminished bilateral breath sounds

Change in mental status

The nurse enters a client's room to assess the older client, who began receiving a blood transfusion 45 minutes earlier, and notes that the client is flushed and dyspneic. On assessment, the nurse auscultates the presence of crackles in the lung bases. The nurse determines that this client most likely is experiencing which complication of blood transfusion therapy? A) Transfusion reaction B) Circulatory (Fluid) overload C) Hypovolemia D) Bacteremia

Circulatory (Fluid) overload

A nurse assess the client after an open lung biopsy. Which assessment finding is matched with the correct intervention? A) Client's respiratory rate is 18 breaths/min-Nurse decreases oxygen flow rate B) Client's HR is 55 beats/min.-Nurse withholds pain medication C) Client has reduced breath sounds-Nurse calls physician immediately D) Client states he is dizzy-Nurse applies oxygen and pulse oximetry

Client has reduced breath sounds-Nurse calls physician immdiately

A nurse is caring for four clients who might be brain dead. Which client would best meet the criteria to allow assessment of brain death? A) Client who is found unresponsive in a remote area of a field by a hunter B) Client with core temperature of 95 F (35 C) for 2 days C) Client with a systolic blood pressure of 92 mm Hg since admission D) Client in a coma for 2 weeks from a motor vehicle crash

Client in a coma for 2 weeks from a motor vehicle crash

A nurse is caring for four clients in the neurologic intensive care unit. After receiving the hand-off report, which client should the nurse see first? A) Client with a Glasgow Coma Scale score that was 9 and is now is 12 B) Client who is requesting pain medication for a headache C) Client with a Glasgow Coma Scale score that was 10 and is now 8 D) Client with a moderate brain injury who is amnesic for the event

Client with Glasgow Coma Scale score that was 10 and is now 8

Which change in the cerebrospinal fluid (CSF) indicates to the nurse that a client may have bacterial meningitis? A) Decreased white blood cells B) Decreased protein C) Cloudy, turbid CSF D) Increased glucose

Cloudy, turbid CSF

A client who was awaiting liver transplantation is excluded from the procedure after the presence of which condition is discovered? A) Hepatic encephalopathy B) Hypertension C) Ascites and shortness of breath D) Colon cancer with metastasis to the liver

Colon cancer with metastasis to the liver

Assessment finding reveal that a client admitted to the hospital has a contact type I hypersensitivity to latex. Which preventive nursing intervention is best in planning care for this client? A) Report the need for desensitization therapy B) Convey the need for pharmacologic therapy to the health care provider C) Discuss symptomatic therapy with the health care provider D) Communicate the need for avoidance therapy to the health care team

Communicate the need for avoidance therapy to the health care team

A client is admitted to the surgical floor with chest pain, shortness of breath, and hypoxemia after having a knee replacement. What diagnostic test dose the nurse expect to help confirm the diagnosis? A) Chest x-ray B) Computed tomography (CT) scan C) Thoracoscopy D) Bronchoscopy

Computed tomography (CT) scan

The following data relate to an older client who is 2 hours postoperative after a cholecystectomy: Skin dry Urine output 20 mL/hr NG tube patent with 100 mL brown drainage/ hr Restless Pulse: 128 beats/min Blood pressure: 88/50 mm Hg Respiratory rate: 20 on ventilator Cardiac output: 2.1 L/min Oxygen saturation: 99% What action by the nurse is best? A) Have respiratory therapy reduce the respiratory rate B) Consult the surgeon about a different antibiotic C) Consult the surgeon about increased IV fluids D) Administer the prescribed pain medication

Consult the surgeon about increased IV fluids

A client is hospitalized and on multiple antibiotics and develops frequent diarrhea. What action by the nurse is most important? A) Delegate frequent perianal care to unlicensed assistive personnel B) Request a prescription for an anti-diarrheal medication C) Place the client on NPO status until the diarrhea resolves D) Consult with the provider about obtaining stool cultures

Consult with the provider about obtaining stool cultures

A nurse assesses a client who is recovering from a total thyroidectomy and notes the development of stridor. Which action should the nurse takes first? A) Contact the provider and prepare for intubation B) Document the finding and assess the client hourly C) Reassure the client that the voice change is temporary D) Place the client in high-Fowler's position and apply oxygen

Contact the provider and prepare for intubation

During an assessment of a 30-year-year old client the nurse notes the reflex shown in the image below. (Babinskis) What actions should the nurse take first? A) Assess bilateral legs for temperature and edema B) Ask the client abut pain in the lower leg and calf C) Document the finding and continue the assessment D) Contact the provider with this abnormal finding

Contact the provider with this abnormal finding

The nurse is caring for a client on a telemetry unit with a regular heart rhythm and rate of 60 beats/min; a P wave precedes each QRS complex, and the PR interval is 0.20 second. Additional vital signs are as follow: blood pressure 118/68 mm Hg, respiratory rate 16 breaths/min, and temperature 98.8 F. Based on the client's clinical presentation, which nursing action is most appropriate at this time? A) Notify the rapid response team B) Administer digoxin C) Continue to monitor D) Administer atropine

Continue to monitor

An emergency nurse cares for a client who is experiencing an acute adrenal crisis. Which order placed by the physician should the nurse question? A) Administer insulin and dextrose B) Administer hydrocortisone succinate (Solu-Cortef) C) Continuous IV infusion of 20 mEq/L potassium chloride in 0.9% sodium chloride D) Assess blood glucose

Continuous IV infusion of 20 mEq/L potassium chloride in 0.9% sodium chloride

The nurse is caring for clients on the medical-surgical unit. What action by the nurse will help prevent a client from have a type II hypersensitivity reactions? A) Correctly identifying the client prior to a blood transfusions B) Keeping the client free of the offending agent C) Providing a latex-free environment for the client D) Administering steroids for severe serum sickness

Correctly identifying the client prior to a blood transfusion

When assessing a client with acute glomerulonephritis, which finding causes the nurse to notify the provider? A) Purulent wound on the leg B) Cola-colored urine C) Crackles throughout the lungs fields D) History of diabetes

Crackles throughout the lung fields

Which assessment indicates to the nurse a therapeutic effect of mannitol (Osmitrol) has been achieved? A) Decreased urine output B) Decreased potassium C) Increased urine osmolaity D) Decreased intracranial pressure

Decreased intracranial pressure

Which clinical manifestation in a client with pyelonephritis indicates that treatment has been effective? A) Increased red blood cell count B) Increased urine specific gravity C) Decreased urine output D) Decreased white blood cells in urine

Decreased white blood cells in urine

The nurse is caring for a client with advanced heart failure who develops asystole. The nurse corrects the graduate nurse when the graduate nurse offers to perform which intervention? A) Administration of epinephrine B) Cardiopulmonary resuscitation (CPR) C) Administration of oxygen D) Defibrillation

Defibrillation

A new graduate RN discovers that her client, who had a tracheostomy placed the previous day, has completely dislodged both the obturator and the tracheostomy tube. Which action should the nurse take first? A) Direct someone to call the Rapid Response Team while using a resuscitation bag and facemask. B) Apply a 100% non-rebreather mask while administering high-flow oxygen C) Auscultate the client;s breath sounds while applying a nasal cannula D) Replace the obturator while reinserting the tracheostomy tube

Direct someone to call the Rapid Response Team while using a resuscitation bag and facemask

A nurse assesses an older adult client who is experiencing a myocardial infarction. Which clinical manifestation can the nurse expect that differs from a younger client's presentation? A) Left lateral chest wall pain B) Disorientation and confusion C) Numbness and tingling of the arm D) Excruciating pain on inspiration

Disorientation and confusion

A client had a percutaneous transluminal coronary angioplasty for peripheral arterial disease. What assessment finding by the nurse indicates a priority outcome for this client has been met? A) Distal pulse on affected extremity are 1+ pre-operative and 3+ post-operative B) Verbalizes understanding of procedure C) Remains on bedrest as directed D) Pain rated as 2/10 after medication

Distal pulse on affected extremity are 1+ pre-operative and 3+ post-operative

A nurse assesses a client with peritonitis. Which clinical manifestation should the nurse expect to find? A) Distended abdomen B) Hyperactive bowel sounds C) Diarrhea D) Bradycardia

Distended abdomen

The patient had an acute ischemic stroke 8 hours ago and has a blood pressure of 170/98. What action should the nurse take? A) Administer an antihypertensive mediation to prevent additional damage B) Document the finding because the increased pressure is needed to perfuse the brain C) Hyperventilate the patient to cause vasodilation D) Teach patient about a low sodium diet

Document the findings because the increased pressure is needed to perfuse the brain

A student nurse is learning about blood transfusion compatibilities. What information does this include? A) Donor blood type AB can donate to anyone B) Donor blood type A can donate to recipient blood type B C) Donor blood type B can donate to recipient blood type O D) Donor blood type O can donate to anyone

Donor blood type O can donate to anyone

A nurse cares for a client with burn injuries form a house fire. The client is not consistently oriented, reports a headache, and has facial flushing. Which action should the nurse take? A) Draw blood for a carboxyhemoglobin level and apply 100% oxygen B) Perform a thorough Mini-Mental State Examination C) Increase the client's intravenous fluid rate D) Increase the client's oxygen and obtain blood gases

Draw blood for a carboxyhemoglobin level and apply 100% oxygen

A client is admitted to the ICU with a medical diagnosis of cerebrovascular accident. The nurse is concerned about the client developing increased intracranial pressure. What assessment finding is key early indication of this occurrence? A) Vomiting B) Hemiplegia C) Drowsiness D) Widened pulse pressure

Drowsiness

A nurse assesses a client with mitral valve stenosis. What clinical manifestation should alert the nurse to the possibility that the client's stenosis has progressed? A) Oxygen saturation of 94% B) Dyspnea on exertion C) Upper extremity weakness D) History of multiple strokes

Dyspnea on exertion

Which manifestation of an oncologic emergency requires the nurse to contact the health care provider immediately? A) Dry cough B) Edema of arms an hands C) New onset of fatigue D) Weight gain

Edema of arms and hands

A client with a diagnosis of diabetic ketoacidosis (DKA) is being treated in an emergency room. Which finding would a nurse expect to note as confirming this diagnosis? A) Elevated blood glucose level and low plasma bicarbonate level B) Decreased urine output C) Comatose state D) Increased respiration and an increase in pH

Elevated blood glucose level and low plasma bicarbonate level

The nurse is caring for a client who sustained superficial partial-thickness burns on the anterior lower legs and anterior thorax. Which of the following does the nurse expect to note during the emergent phase of the burn injury? A) Increased urinary output B) Elevated hematocrit levels C) Decreased heart rate D) Increased blood pressure

Elevated hematocrit levels

The nurse is caring for a client with a third degree burns of the anterior and posterior chest. The client begins taking shallow breaths and says that his "chest is being crushed." What intervention should the nurse expect for this patient? A) Emergent tracheotomy B) Emergent escharotomy C) Epinephrine administration D) Morphine administration

Emergent escharotomy

A client in the hospital and receiving IV antibiotics. When the nurse answers the client's call light, the client presents an appearance as shown below: (swelling in the face). What action by the nurse takes priority? A) Ensure a patent airway while calling the Rapid Response Team B) Reassure the client that these manifestations will go away C) Apply oxygen by facemask at 100% and a pulse oximeter D) Administer epinephrine 1:1000, 0.3 mg IV push immediately

Ensure a patent airway while calling the Rapid Response Team

A hospitalized client is placed on Contact Precautions. The client needs to have a computed tomography (CT) scan. What action by the nurse is most appropriate? A) Travel with the client and continue to wear a gown while transporting the patient B) Notify the physician that the client cannot leave the room for the CT scan C) Ensure that the radiology department is aware of the isolation precautions D) No special precautions are needed when this client leaves the unit

Ensure that the radiology department is aware of the isolation precautions

A nurse is preparing to admit a client on mechanical ventilation from the emergency department. What action by the nurse takes priority? A) Planning to suction the client upon arrival to the room B) Obtaining personal protective equipment C) Ensuring there is a bag-valve-mask in the room D) Assessing that the ventilator settings are correct

Ensuring there is a bag-valve-mask in the room

The nurse is caring for the client in the emergency department following a head injury. The client momentarily lost consciousness at the time of the injury and then regained it. The client now has lost consciousness again. The nurse takes quick action, knowing that this is compatible with: A) Concussion B) Skull Fracture C) Subdural Hematoma D) Epidural Hematoma

Epidural Hematoma

A client hospitalized for chemotherapy has a hemoglobin of 6.1 mg/dL. What medication should the nurse prepare to administer? A) Filgrastim (Neupogen) B) Mesna (Mesnex) C) Epoetin alfa (Epogen) D) Oprelvekin (Neumega)

Epoetin alfa (Epogen)

A client is taking isoniazid, rifampin, pyrazinamide, and ethambutol for tuberculosis. The client calls to report visual changes, including blurred vision and reduced visual fields. Which medication may be causing these changes? A) Isoniazid B) Pyrazinamide C) Rifampin D) Ethambutol

Ethambutol

An emergency room nurse initiates care for a client with a cervical spinal cord injury who arrives via emergency medical services. Which action should the nurse take first? A) Assess level of consciousness B) Evaluate respiratory status C) Obtain vital signs D) Administer oxygen therapy

Evaluate respiratory status

A patient on prolonged mechanical ventilation is given nutritional support. He has failed weaning repeatedly and his carbon dioxide levels are markedly elevated. Which of the following may be responsible for the elevated carbon dioxide levels? A) Excessive nitrogenous amino acids B) Excessive cholesterol C) Excessive carbohydrates D) Excessive long chain fatty acids

Excessive carbohydrates

A client recently diagnosed with human immune deficiency virus (HIV) is being treated for candidiasis. Which medication does the nurse anticipate the health care provider will prescribe for this client? A) Trimethoprim/sulfamethoxazole (Bactrim) B) Fluconazole (Diflucan) C) Rifampin (Rifadin) D) Acyclovir (Zovirax)

Fluconazole (Diflucan)

A nurse reviews the following data in the chart of a client with burn injuries: Bilateral leg burns present with a white and leather-like appearance. No blisters or bleeding present. Client rates pain 2/10 on a scale of 0-10. A) Partial-thickness deep B) Superficial C) Partial-thickness superficial D) Full thickness

Full thickness

A nurse assesses bilateral wheezes in a client with burn injuries inside the mouth. Four hours later the wheezing is no longer heard. Which action should the nurse take? A) Gather appropriate equipment and prepare for an emergency airway B) Administer humidified oxygen C) Document the findings and reassess in 1 hour D) Loosen any constrictive dressings on the chest

Gather appropriate equipment and prepare for an emergency airway

A 19-year-old man has been having trouble breathing for a week. He has occasional hemoptysis, decreased urine output, hypertension, and tachycardia. Further testing reveals that he has areas of consolidation over his lung fields (by chest x-ray) and glomerulonephritis with reduced kidney function. The nurse recognizes that these symptoms are associated with that condition? A) Anaphylactic reaction to an unknown allergen B) Sjorgren's syndrome C) Goodpasture's syndrome D) Grave's disease

Goodpasture's syndrome

A client is started on furosemide (Lasix) for heart failure. The nurse instructs the client about the drug and tells the client to notify the physician immediately if which symptoms occur? A) Tachycardia and hypertension B) Heart palpitations and confusion C) Nausea, vomiting, and visual changes D) Headache

Heart palpitations and confusion

An emergency room nurse admitted a client experiencing a thyroid storm. The client is likely exhibiting all of the following signs and symptoms except: A) Intolerance to heat B) Respirations of 42 breaths per minute C) Restlessness D) Heart rate of 20 beats per minute

Heart rate of 20 beats per minute

A client is admitted to the hospital with suspected Goodpasture's syndrome. Which findings does the nurse expect to observe? A) Weight loss B) Hemoptysis C) Increased urine output D) Bradycardia

Hemoptysis

A client undergoes an esophagogastroduodenoscopy (EDG). The proceduralist explains that bulging, engorged blood vessels were noted at the base of the esophagus. The nurse understands that this finding places the client at an increased risk of: A) Hemorrhage B) Hepatic encephalopathy C) Hemorrhoids D) Cirrhosis

Hemorrhage

A client sustains deep partial-thickness burns to the arms and anterior chest. It has been 8 hours since admission. All of the following are assessment findings at this time. Which one of the following should have the highest priority for intervention? A) Diminished radial pulses B) Decreased urine output C) Hoarseness and stridor D) Increased restlessness and irritability

Hoarseness and stridor

The nurse caring for a client who is receiving intravenous (IV) magnesium sulfate. Which assessment parameter is critical? A) Asking the client about feeling depressed B) Monitoring of serum calcium levels C) Monitoring 24-hour urine output D) Hourly deep tendon reflexes (DTRs)

Hourly deep tendon reflexes (DTRs)

A client has had a unilateral adrenalectomy to remove a tumor. To prevent complications, which problem for the client this takes priority? A) Absent bowel sounds B) Hypotension C) Increased temperature D) Increased urine output

Hypotension

A client with acquired immune deficiency is seen in the clinic for re-evaluation of the immune system's response to prescribed medication. Which test result does the nurse convey to the health care provider? A) Therapeutic highly active antiretroviral therapy (HAART) level B) Improved CD4+ T-cell count and reduce viral load C) Positive Papanicolaou (Pap) test D) Positive human immune deficiency virus (HIV), enzyme-linked immunosorbent assay (ELISA)

Improved CD4+ T-cell count and reduced viral load

Which laboratory result indicates that fluid restrictions have been effective in treating syndrome of inappropriate antidiuretic hormone (SIADH)? A) Decreased hematocrit B) Decreased serum osmolality C) Increased serum sodium D) Increased urine specific gravity

Increased serum sodium

The nurse is caring for a client on the medical-surgical unit who suddenly becomes unresponsive and has not pulse. The cardiac monitor shows the rhythm below (small jagged humps). After calling for assistance and a defibrillator, which action should the nurse take next? A) Perform a pericardial thump B) Initiate cardiopulmonary resuscitation (CPR) C) Administer 1 mg IV Epinephrine D) Start an 18-gauge intravenous line

Initiate cardiopulmonary resuscitation (CPR)

An emergency department nurse assesses a client with kidney trauma and notes that the client's abdomen is tender and distended and blood is visible at the urinary meatus. Which prescription should the nurse consult the provider about before implementation? A) Assessing vital signs every 15 minutes B) Inserting an indwelling urinary catheter C) Administering intravenous fluids at 125 mL/hr D) Typing and crossmatching for blood products

Inserting an indwelling urinary catheter

A nurse is assisting the health care provider who is intubating a client. The provider has been attempting to intubate for 40 seconds. What action by the nurse takes priority? A) Monitor the client's oxygen saturation B) Find another provider to intubate C) Ensure the client has adequate sedation D) Interrupt the procedure to give oxygen

Interrupt the procedure to give oxygen

A recently admitted client who is in sickle cell crisis requests "something for pain." What does the nurse administer? A) Oral ibuprofen (Motrin) B) Intravenous (IV) hydromorphone (Dilaudid) C) Intramuscular (IM) morphine sulfate D) Oral morphine sulfate (MS-Contin)

Intravenous (IV) hydromorphone (Dilaudid)

A client is admitted to a hospital with a diagnosis of diabetic ketoacidosis (DKA). The initial blood glucose was 800 mg/dL. A continuous intravenous infusion of regular insulin is initiated, along with intravenous rehydration with normal saline. Serum glucose level is now 260 mg/dL. The nurse will expect to prepare to administer which of the following? A) One ampule of 50% dextrose B) NPH insulin subcutaneously C) Phenytoin (Dilantin) for the prevention of seizures D) Intravenous fluids containing 5% dextrose

Intravenous fluids containing 5% dextrose

The nurse is reviewing orders for a client with possible esophageal trauma after a car crash. Which request does the nurse implement first? A) Give total parenteral nutrition (TPN) through a central venous catheter B) Keep the client nothing by mouth (NPO) for possible surgery C) Administer cefazolin (Kefzol) 1 g intravenously D) Obtain a computed tomography (CT) scan of the chest and abdomen

Keep the client nothing by mouth (NPO) for possible surgery

A nurse is caring for several clients at risk for shock. Which laboratory value requires the nurse to communicate with the health care provider? A) White blood cell count: 11,000/mm3 B) Lactate: 6 mmol/L C) Sodium: 150 mEq/L D) Creatinine: 0.9 mg/dL

Lactate: 6 mmol/L

A client admitted with a deep vein thrombosis (DVT). Which intervention would be most appropriate for pain relief? A) Leg elevation B) Immobilization of the affected leg C) Application of heat D) Massage

Leg elevation

A client is returned to the unit following a percutaneous cardiac interventions (PCI). The access site was the cleint's right femoral artery. Which nursing action is appropriate at this time? A) Restrict fluid intake and encourage ambulation B)Limit motion of the affected extremity and assess the puncture site C) Apply heat to the puncture site and passively exercise the involved extremity D) Discourage fluid intake and place the client in the prone position

Limit motion of the affected extremity and assess the puncture site

A client recently admitted to your unit reports consuming at least on handle of vodka each day. He is beginning to experience nausea, tremors, and extreme agitation. What medication do you expect the ordering physician to prescribe? A) Midazolam (Versed) B) Lorazepam (Ativan) C) Propofol (Diprivan) D) Morphine Sulfate (Duramorph)

Lorazepam (Ativan)

A nurse cares for a client who is experiencing status epilepticus. Which prescribed medication should the nurse prepare to administer? A) Atenolol (Tenormin) B) Phenytoin (Dilantin) C) Lisinopril (Prinivil) D) Lorazepam (Ativan)

Lorazepam (Ativan)

Which clinical symptoms in a postoperative client indicate early sepsis with an excellent recovery rate if treated? A) Reduced urinary output an increased respiratory rate B) Low-grade fever and mild hypotension C) Low oxygen saturation rate and decreased cognition D) Localized erythema and edema

Low-grade fever and mild hypotension

A client with respiratory failure has been intubated and placed on a ventilator and is requiring 100% oxygen delivery to maintain adequate oxygenation. Twenty-four hours later, the nurse notes new-onset crackles and decreased breath sounds, and the most recent arterial blood gases (ABGs) show a PaO2 level of 95 mm Hg. The ventilator is not set to provide positive end-expiratory pressure (PEEP). Why is the nurse concerned? A) The low PaO2 level may result in oxygen toxicity B) Lung sounds may indicate absorption atelectasis C) The level of oxygen delivery may indicate absorption atelectasis D) The 100% oxygen delivery requirement indicates immediate extubation

Lung sound may indicate absorption atelectasis

A nurse plans care for a client with acute pancreatitis. Which intervention should the nurse include in this client's care to reduce discomfort? A) Maintain nothing by mouth (NPO) and administer intravenous fluids B) Administer morphine sulfate intravenously every 12 hours as needed C) Place the client in semi-Fowler's position with the head of bed elevated D) Provide small, frequent feedings with no concentrated sweets

Maintain nothing by mouth (NPO) and administer intravenous fluids

The nurse is preparing to care for a client receiving peritoneal dialysis. Which of the following would be included in the nursing plan of care to prevent the major complication associated with peritoneal dialysis? A) Maintain strict aseptic technique B) Add heparin to the dialysate solution C) Change the catheter site dressing daily D) Monitor the client's level of consciousness

Maintain strict aseptic technique

A patient with type 1 diabetes mellitus has been ordered insulin aspart (Novolog) 10 units at 7:00 AM. what nursing intervention will the nurse perform after administering this medication? A) Flush the IV B) Perform a fingerstick blood sugar test C) Have the patient void and dipstick the urine D) Make sure the patient eats breakfast immediately

Make sure the patient eats breakfast immediately

The nurse is monitoring a postoperative craniotomy client with increased intracranial pressure (ICP). Which pharmacologic agent does the nurse expect to be requested to maintain the ICP within a specified range? A) Phenytoin (Dilantin) B) Dexamethasone (Decadron) C) Mannitol (Osmitrol) D) Hydrochlorothiazide ()

Mannitol (Decadron)

A nurse reviews the medication list of client recovering from a computed tomography (CT) scan with IV contrast to rule out small bowel obstruction. Which medication should alert the nurse to contact the provider and withhold the prescribed dose? A) Pioglitazone (Actos) B) Glimepiride (Amaryl) C) Metformin (Glucophage) D) Glipizide (Glucotrol)

Metformin (Glucophage)

A nurse is caring for a cancer patient receiving subcutaneous morphine sulfate for pain. Which of the following nursing actions is most important in the care of this patient? A) Monitor temperature B) Monitor respiratory rate C) Monitor urine output D) Monitor heart rate

Monitor respiratory rate

A client with terminal pancreatic cancer is near death and reports increasing shortness of breath with associated anxiety. Which hospice protocol order does the nurse implement firs? A) Prednisone (Deltasone) elixir 10 mg orally B) Albuterol (Proventil) 0.5% solution per nebulizer C) Morphine sulfate (Roxanol) 5 to 10 mg sublingually as needed D) Oxygen 2 to 6 L/min per nasal cannula

Morphine sulfate (Roxanol) 5 to 10 mg sublingually as needed

The nurse teaches a client who is recovering from acute kidney disease to avoid which type of medication? A) Nonsteroidal anti-inflammatory drugs (NSAIDs) B) Angiotensin-converting enzyme (ACE) inhibitors C) Opiates D) Acetaminophen

Nonsteroidal anti-inflammatory drugs (NSAIDs)

A client is receiving an infusion of alteplase (Activase) for an intra-arterial clot. The client begins to mumble and is disoriented. What action by the nurse takes priority? A) Prepare to administer vitamin K B)Notify the Rapid Response Team C) Turn down the infusion rate D) Place the patient in high fowler's position

Notify the Rapid Response Team

The nurse is performing an assessment on a client who has returned from the dialysis unit following hemodialysis. The client is complaining of headache and nausea and is extremely restless. Which of the following is the most appropriate nursing action? A) Medicate the client for nausea B) Elevate the head of the bed C) Notify the physician D) Monitor the client

Notify the physician

A client in the intensive care unit scheduled for a lumbar puncture (LP) today. On assessment, the nurse finds the client breathing irregularly with one pupil fixed and dilated. What action by the nurse is best? A) Notify the provider of the findings immediately and prepare for cancellation of the lumbar puncture B) Document these findings in the client's record and continue to monitor the patient C) Notify the provider and prepare the client for an emergent lumbar puncture D) Give the prescribed preprocedure sedation and ensure that the informed consent is signed

Notify the provider of the findings immediately and prepare for cancellation of the lumbar puncture

After nurse cares for a client with a fractured fibula. Which assessment should alert the nurse to take immediate action? A) Numbness in the extremity B) Feeling cold while lying in bed C) Swollen extremity at the injury site D) Pain of 4 on a scale of 0 to 10

Numbness in the extremity

A client who is immune deficiency virus (HIV) positive and has a CD4+ count of 15 has been admitted with a fever and abdominal pain. Which health care provider request does the nurse implement first? A) Obtain a 12-lead electrocardiogram (ECG) B) Give cefazolin (Kefzol) 500 mg IV C) Call for a portable chest x-ray D) Obtain blood cultures from two sites

Obtain blood cultures from two sites

A client a history of esophageal varices has just been admitted to the emergency department after vomiting a large quantity of blood. Which action does the nurse take first? A) Obtain the charts form the previous admission B) Obtain pulse and blood pressure C) Ask about abdominal pain D) Listen for bowel sounds in all quadrants

Obtain pulse and blood pressure

A 71-year-old male patient who is currently undergoing coronary artery bypass graft (CABG) surgery has just experienced intraoperative vomiting. The nurse should consequently anticipate the use of which of the following drugs? A) Fentanyl (Sublimaze) B) Ondansetron (Zofran) C) Midazolam (Versed) D) Meperidine (Demerol)

Ondansetron (Zofran)

A client's is connected to a continuous ECG monitor. Which component of their ECG represents proper conduction of the elctrical impulse from the atria to the ventricles? A) QRS complex B) P wave C) ST segment D) PR interval

PR interval

A client admitted with severe dyspnea and diaphoresis is diagnosed with acute respiratory distress syndrome. The nurse would expect which result from the client's diagnostic tests and physical assessment? A) Pulmonary capillary wedge pressure [PCWP] greater than 18 mm Hg B) Abnormal lung sounds heard on auscultation C) Large, black areas noted on the client's chest x-ray D) PaO2 less than 60 mm Hg despite use of higher oxygen levels

PaO2 less than 60 mm Hg despite use of higher oxygen levels

The nurse is caring for the client who has undergone renal angiography using the left femoral artery for access. The nurse determines that the client is experiencing a complication of the procedure if which of the following is observed? A) Urine output, 40 mL/hr B) Blood pressure, 108/66 C) Pallor and coolness of the left leg D) Absence of hematoma in the left groin

Pallor and coolness of the left leg

A nurse assesses a client with a spinal cord injury at level T5. The client's blood pressure is 210/106 mm Hg, and the client presents with a flushed face and blurred vision. Which action should the nurse take first? A) Place the client in a supine position B) Initiate oxygen via a nasal cannula C) Administer a prescribed beta blocker D) Palpate the bladder for distention

Palpate the bladder for distention

A nurse cars for a client who has a moderate amount of frank blood in his stool. The nurse suspects that the client is experiencing a lower gastrointestinal bleed. All of the following are risk factors for a lower GI bleed except: A) Peptic ulcer disease B) Diverticulosis C) Crohn's disease D) Colon cancer

Peptic ulcer disease

A client is receiving total parenteral nutrition (TPN). What action by the nurse is most important? A) Checking the TPN with another nurse B) Changing the IV dressing each day C) Performing appropriate hand hygiene D) Assessing blood glucose as directed

Performing appropriate hand hygiene

Which of the following interventions or medications is the emergency treatment for cardiac tamponade? A) CABG B) Pericardiocentesis C) Dopamine D) Percutaneous coronary intervention (PCI)

Pericardiocentesis

Which intervention best assists the client with acute pulmonary edema in reducing anxiety and dyspnea? A) Reassure the client that his distress can be relieved with proper intervention B) Place the client in high-Fowler's position with the legs down C) Monitor pulse oximetry and cardiac rate and rhythm D) Ask a family member to remain with the client

Place the client in high-Fowler's position with the legs down

A client with human immune deficiency virus is admitted to the hospital with fever, night sweats, and severe cough. Laboratory results include a CD4+ cell count of 180/mm3 and a negative tuberculosis (TB) skin test 4 days ago. What action should the nurse take first? A) Place the client under Airborne Precautions B) Initiate Droplet Precautions for the cleint C) Notify the provider about the CD4+ results D) Use Standard Precautions to provide care

Place the client under Airborne Precautions

While suctioning a patient, vagal stimulation occurs. What is the appropriate nursing action? A) Instruct the pt. to breathe slowly and deeply B) Oxygenate the pt. with 100% oxygen C) Instruct the pt. to cough D) Place the pt. in high Fowler's position

Place the pt. in high Fowler's position

A client has an acute myocardial infarction. What assessment finding indicates to the nurse that a significant complication has occurred? A) Urine output of 40 mL/hr for 4 hours B) Blood pressure that is 10 mm Hg below baseline C) Poor peripheral pulses and cool skin D) Oxygen saturation of 94% on room air

Poor peripheral pulses and cool skin

A nurse is evaluating the status of the client who had a craniotomy 3 days ago. The nurse would suspect that the client is developing meningitis as a complication of surgery if the client exhibits: A) Glasgow Coma Scale score of 15 B) Positive Romberg sign C) Positive Brudzinski's sign D) Negative Kernig's sign

Positive Brudzinski's sign

Which problem for a client with cirrhosis takes priority? A) Potential for injury related to hemorrhage B) Insufficient knowledge relate to the prognosis of the disease process C) Inadequate nutrition related to an inability to tolerate usual dietary intake D) Discomfort related to the progression of the disease process

Potential for injury related to hemorrhage

A client's 12 lead electrocardiogram (ECG) is shown in the rhythm strip provided. The nurse correctly interprets the ECG strip as: (regular looking rhythm with the exception of occasional large looking QRS) A) Premature atrial contractions (PACs) which are a prelude to lethal arrhythmias. B) Ectopic beat which likely indicate myocardial infarction C) Normal sinus rhythm (NSR) with no variation D) Premature ventricular contractions (PVCs) which are often benign

Premature ventricular contractions (PVCs) which are often benign

A client is assessed by the nurse after a hemodialysis session. The nurse notes bleeding form the client's nose and around the intravenous catheter. What action by the nurse is the priority? A) Hold pressure over the client's nose for 10 minutes B) Take the client's pulse, blood pressure, and temperature C) Assess for a bruit or thrill over the arteriovenous fistula D) Prepare protamine sulfate for administration.

Prepare protamine sulfate for administration

A nurse is caring for a client who has methicillin-resistant Staphylococcus aureus (MRSA) infection cultured from the urine. What action by the nurse is most appropriate? A) Wash hands only after taking off gloves after care B) Wear a respirator when handling urine output C) Wear a surgical mask at all times D) Prepare to administer vancomycin (Vancocin)

Prepare to administer vancomycin (Vancocin)

A client with a history of atrial fibrillation is receiving sodium heparin 24 hours after receiving thrombolytic therapy for a stroke. Which emergency drug does the nurse ensure is on the floor? A) Physostigimine B) Phytonadione (Vitamin K) C) Protamine sulfate D) Naloxone (Narcan)

Protamine sulfate

When taking the health history of a client with acute glomerulonephritis (GN), the nurse questions the client about which related cause of the problem? A) Recent respiratory infection B) Hypertension C) Unexplained weight loss D) Neoplastic disease

Recent respiratory infection

The nurse assumes care for a patient who is currently receiving a dose of intravenous vancomycin (Vancocin) infusing at 20 mg/min. The nurse notes red blotches on the patient's face, neck, and chest and assesses a blood pressure of 80/55 mm Hg. What is the patient likely experiencing? A) Anaphylactic Reaction B) Stevens-Johnson Syndrome C) Red-man Syndrome D) Vancomycin Toxicity

Red-man syndrome

In an intensive care unit, your client is on Assist-control (AC) ventilation with a preset tidal volume of 750mL and a respiratory rate of 14 breaths/minute. While assessing the client, you note that the client is breathing more than 14 breaths/minute. what is this patient most likely to experience? A) Increase flow rate B) Respiratory alkalosis C) Respiratory acidosis D) Decreased FiO2

Respiratory alkalosis

The nurse assesses multiple clients who are receiving transfusions of blood components. Which assessment indicates the need for the nurse's immediate action? A) A partial thromboplastin time (PTT) that is 1.2 times normal in a client who received a transfusion of fresh-frozen plasma (FFP) B) Temperature of 99.1 F (37.3 C) for a client with a platelet transfusion C) Sleepiness in a client who received diphenhydramine (Benadryl) as a premedication D) Respiratory rate of 36 breaths/min in a client receiving red blood cells

Respiratory rate of 36 breaths/min in a client receiving red blood cells

In the care of a client with acute pancreatitis, which assessment parameter requires immediate nursing intervention? A) Serum glucose of 136 mg/dL B) Heart rate of 105 beats/min C) Respiratory rate of 43 breaths/min D) Blood pressure of 102/76 mm Hg

Respiratory rate of 43 breaths/min

The nurse is preparing to care for a burn client scheduled for an escharotomy procedure for a third-degree circumferential arm burn. The nurse understands that the anticipated therapeutic outcome of the escharotomy is: A) Formulation of granulation tissue B) Brisk bleeding from the site C) Return of distal pulses D) Decreasing edema formation

Return of distal pulses

The nurse is assessing a client who is experiencing an acute episode of cholecystitis. Where should the nurse anticipate the location of the pain? A) Right upper quadrant, radiating to the right scapula and shoulder B) Left lower quadrant, radiating to the umbilicus C) Left upper quadrant, radiating to the left scapula and shoulder D) Right lower quadrant, radiating to the back

Right upper quadrant, radiating to the right scapula and shoulder

A nursing assessment confirms that a client, being evaluated for a cardiac-related diagnosis, is experiencing weigh gain, pitting bilateral low extremity edema, nausea, and decreased urine output. This nurse suspects that the client may be experiencing: A) Right-sided heart failure B) Left-sided heart failure C) Angina pectoris D) Cardiomyopathy

Right-sided heart failure

A nurse is working to decrease the number of health care acquired infections (HCAIs) on her unit. Based on the nurse's knowledge of the most frequent cause of HCAIs, what does the nurse choose to address first? A) Scheduled and PRN central line dressing changes B) Frequent pulmonary toileting C) Scheduled and PRN perineal and foley catheter care D) Wound assessment, cleaning, and care

Scheduled and PRN perineal and foley catheter care

The client has clear fluid leaking from the nose following a basilar skull fracture. The nurse assesses that this is cerbrospinal fluid if the fluid: A) Is clear and test negative for glucose B) Clumps together on the dressing and has a pH of 7 C) Is grossly bloody in appearance and has a pH of 6 D) Separates into concentric rings on gauze and tests positive for glucose

Separates into concentric rings on gauze and tests positive for glucose

A nurse assesses a client who has appendicitis. Which clinical manifestation should the nurse expect to find? A) Severe, steady right lower quadrant pain B) Generalized abdominal pain associated with nausea and vomiting C) Marked peristalsis and hyperactive bowel sounds D) Abdominal pain that increases with knee flexion

Severe, steady right lower quadrant pain

A nurse teaches a client with diabetes mellitus about differentiating between hypoglycemia and ketoacidosis. The client demonstrates an understanding of teaching by stating that glucose will be taken if which of the following symptoms of hypoglycemia develop? A) Shakiness B) Fruity breath odor C) Blurred vision D) Polyuria

Shakiness

Which assessment finding requires immediate nursing intervention in a client with severe ascites? A) Oral temperature, 100.8 F B) Tachycardia, rate 110 bpm C) Shallow respirations, rate 32 breaths/min D) Confusion

Shallow respirations, rate 32 breaths/min

A nurse administers prescribe adenosine (Adenocard) to a client. Which response should the nurse assess for as the expected therapeutic response? A) Short period of asystole B) Increased heart rate C) Decreased nausea D) Hypertensive crisis

Short period of asystole

A client is placed on a medical regimen of doxorubicin (Adriamycin), cyclophosphamide (Cytoxan), and fluorouracil (5-FU) for breast cancer. Which side effects seen in the client should the nurse report to the provider immediately? A) Shortness of breath B) Nausea and vomiting C) Hair loss D) Mucositis

Shortness of breath

The nurse is caring for a client diagnosed with acute respiratory distress syndrome (ARDS) considers that, in this client, impaired gas exchange is mostly likely related to which of the following? A) Excessive alpha-1-antitrypsin B) Air trapping in the alveoli C) Shunting of blood around non-ventilated aveoli D) Thick secretions block the airways

Shunting of blood around non-ventilated alveoli

A nursing student is caring for a client who is experiencing an acute sickle cell crisis. Which of the following statements made by the student demonstrates an accurate understanding of this disease process? A) Sickled cells increase blood flow through the body and cause a great deal of pain B) Sickled cells are unable to flow easily through the microvasculature and their clumping obstructs blood flow C) Bone marrow depression occurs because of the development of sickled cells D) Sickled cells mix with unsickled cells and cause the immune system to become depressed

Sickled cells are unable to flow easily through the microvasculature and their clumping obstructs blood flow

A client with acute kidney injury has a blood pressure of 76/55 mm Hg. The health care provider ordered 1000 mL of normal saline to be infused over 1 hour to maintain perfusion. The client is starting to develop shortness of breath. What is the nurse's priority action? A) Take the client's pulse B) Ask for insertion of a pulmonary artery catheter C) Slow down the normal saline infusion D) Calculate the mean arterial pressure (MAP)

Slow down the normal saline infusion

A nurse assess a client with atrial fibrillation. Which manifestation should alert the nurse to the possibility of a serious complication from this condition? A) Speech alterations B) Fatigue C) Sinus tachycardia D) Hypertension

Speech alterations

The nurse is assessing a client with a cardiac infection. Which symptoms support the diagnosis of infective endocarditis instead of pericarditis or rheumatic carditis? A) Friction rub auscultated at the left lower sternal border B) Pain aggravated by breathing, coughing, and swallowing C) Thickening of the endocardium D) Splinter hemorrhages

Splinter hemorrhages

A nurse is caring for a 55-year-old client with cirrhosis and ascites. The client develops a rigid, tender abdomen and an oral temperature of 102 F. What complication of ascites is the client most likely experiencing? A) Biliary obstruction B) Hemorrhage C) Hepatic encephalopathy D) Spontaneous bacterial peritonitis (SBP)

Spontaneous bacterial peritonitis (SBP)

A nurse is assessing a client who has a tracheostomy. the nurse notes that the tracheostomy tube is pulsing with the heartbeat as the client's pulse is being taken. No other abnormal findings are noted. What action by the nurse is most appropriate? A) No action is needed at the time; this is a normal finding in some clients B) Remove the tracheostomy tube; ventilate the client with a bag-valve-mask C) Call the operating room to inform them of a pending emergency case D) Stay with the client and have someone else call the provider immediately

Stay with the client and have someone else call the provider immediately

Respirations of a sedated client with a new tracheostomy have become noisy, and the ventilator alarms indicate high peak pressures. The ventilator tube is clear. What is the best immediate action by the nurse? A) Removing the inner cannula of the tracheostomy B) Humidifying the oxygen source C) Increasing oxygenation D) Suctioning the client

Suctioning the client

A client states "It feels like my heart is beating out of my chest." The nurse looks at the client's ECG monitor and notes the rhythm shown in the ECG strip below. How does the nurse interpret this rhythm strip? A) Sinus Tachycardia B) Ventricular Fibrillation C) Ventricular Tachycardia D) Supraventricular Tachycardia

Supraventricular Tachycardia

The nurse is caring for postoperative clients at risk for hypovolemic shock. Which condition represents an early symptom of shock? A) Heart blocks B) Bradypnea C) Tachycardia D) Hypotension

Tachycardia

A nurse is performing an admission assessment on a client admitted with a diagnosis of pheochromocytoma. The nurse assesses for the major symptom associated with pheochromocytoma when the nurse: A) Palpates the skin for its temperature B) Takes the client's blood pressure C) Obtains the client's weight D) Tests the client's urine for glucose

Takes the client's blood pressure

The client had an ischemic stroke and is undergoing rehabilitation. He is diagnosed with homonymous hemianopsia. What should the nurse do? A) Better arrange the environment to suit the patient's needs B) Teach the patient to turn his head to scan the environment C) Have the patient wear an eye patch D) Obtain prescriptive glasses for the patient

Teach the patient to turn his head to scan the environment

Which sign/symptom is essential for the nurse to report to the provider when caring for a client with Raynaud's phenomenon? A) The affected extremity becomes purple and cold B) The client's extremity became white, then red temporarily C) The client states that the digits are painful when they are white D) Nifedipine (Procardia) administration caused the blood pressure to change from 134/76

The affected extremity becomes purple and cold

A nurse evaluates a client with acute glomerulonephritis (GN). Which manifestation should the nurse recognize as a positive response to the prescribed treatment? A) The client has lost 11 pounds in the past 10 days B) The client's urine specific gravity is 1.048 C) No blood is observed in the client's urine D) The client's blood pressure is 152/88 mm Hg

The client has lost 11 pounds in the past 10 days

A nurse assesses a client with the National Institutes of Health (NIH) Stroke Scale and determines the client's score to be 36. How should the nurse plan care for this client? A) The client will need cuing only B) The client will need safety precautions C) The client will need near-total care D) The client will be discharged home

The client will need near-total care

A client arrives to the emergency department via ambulance following a motor vehicle accident. The client is intubated and unresponsive at this time. Based on the image below, the nurse knows that the client's burns are: A) Second degree, deep partial thickness injury B) Third degree, full thickness injury C) First Degree, partial thickness injury D) Second degree, superficial thickness injury

Third degree, full thickness injury

A client with a stroke is being evaluated for fibrinolytic therapy. What information from the client or family is most important for the nurse to obtain? A) Time of symptom onset B) Progression of symptoms C) Loss of bladder control D) Other medical conditions

Time of symptoms onset

A client is brought to the emergency room having experienced blood loss related to an arterial laceration. Fresh frozen plasma (FFP) is ordered and transfused to replace fluid and blood loss. The nurse understands that the rationale for transfusing FFP in this client is: A) To promote rapid volume expansion B) To decrease oozing of blood from puncture sites C) To increase the hemoglobin and hematocrit levels D) To teat the loss of platelets

To promote rapid volume expansion

A patient who has been taking nitroglycerin for angina has developed variant angina, and the provider has added verapamil (Calan) to the patient's regimen. The nurse will explain that verapamil is given for which purpose? A) To constrict systemic vessels B) To increase cardiac contractility C) To relax coronary arteries D) To improve renal perfusion

To relax coronary arteries

The client arrives at the emergency department with complaints of low abdominal pain and hematuria. The client is afebrile. The nurse next assesses the client to determine a history of: A) Trauma to the bladder or abdomen B) Glomerulonephritis C) Pyelonephritis D) Renal cancer in the client's family

Trauma to the bladder or abdomen

The nurse is infusing packed red blood cells into a patient with anemia when the patient reports a backache and chills. The nurse notes hypotension. Which type of hypersensitivity reactions is occurring? A) Type IV, cell-mediated hypersensitivity B) Type I, anaphylactic hypersensitivity C) Type III, immune complex hypersensitivity D) Type II, cytotoxic hypersensitivity

Type II, cytotoxic hypersensitivity

A nurse is preparing to administer heparin when the client states, "I had a bad reaction to heparin once. I think they call it HIT." From this information the nurse is aware that the medication to which the client reacted was most likely of which form? A) Fractionated heparin B) Low-molecular weight heparin C) Lovenox D) Unfractionated heparin

Unfractionated heparin

A nurse is caring for a postoperative 70-kg client who had major blood loss during surgery. Which findings by the nurse should prompt immediate action to prevent acute kidney injury? A) Urine output of 100 mL in 4 hours B) Urine output of 500 mL in 12 hours C) Amber, odorless urine D) Blood pressure of 115/75 mm Hg

Urine output of 100 mL in 4 hours

A nurse assesses a client who is recovering from a paracentesis 1 hour ago. Which assessment finding requires action by the nurse? A) Blood pressure increases from 110/58 to 120/62 mm Hg B) Respiratory rate decreases from 18 to 14 breaths/min C) A decrease in the client's weight by 6 kg D) Urine output via indwelling urinary catheter in 20 mL/hr

Urine output via indwelling urinary catheter is 20 mL/hr

An ICU nurse is caring for a client with known esophageal varices. During change of shift the client calls out for the nurse. Upon entering the room, the nurse sees a large basin filled to the top with bright red emesis. The client becomes increasingly lethargic over the next few minutes. The nurse calls a rapid response and the client is quickly intubated. The client's provider is at the bedside and insets a Sangstaken-Blakemore tube. Based on the nurse's understanding of this type of tube, the nurse expects that further treatment for this client will include: A) Administration of PRBCs until bleeding stops and the tube may be removed B) Gastric suction and iced saline lavage C) Whipple procedure D) Variceal band ligation

Variceal band ligation

The nurse has an order to administer 50% oxygen to a client with pulmonary edema. The nurse would select which of the following oxygen administration systems that allow that percentage of oxygen to be delivered? A) Partial Rebreather B) Nonrebreather Mask C) Nasal Cannula D) Venturi Mask

Venturi Mask

A nurse evaluates prescriptions for a client with chronic atrial fibrillation. Which medication should the nurse expect to find on this client's medication administration record to prevent a common complication of this condition? A) Atropine (Sal-Tropine) B) Lidocaine (Xylocaine) C) Sotalol (Betapace) D) Warfarin (Coumadin)

Warfarin (Coumadin)

A client has an external ventricular device (EVD). What action by the nurse is most important to do at the beginning of the shift? A) Monitoring the client's phlebostatic axis B) Emptying the CSF from the drainage bag C) Handling the fiberoptic cable with care to avoid breakage D) Zeroing and re-leveling the device

Zeroing and re-leveling the device


Related study sets

Abeka Science 9th Grade, Chapter 12

View Set

Medical-Surgical: Immune and Infectious Quiz

View Set

SYSTEMS ANALYSIS & DESIGN Chapter 9

View Set

Peds Final Exam Practice Questions

View Set

Ch 14 Accounts Payable and Other Liabilities

View Set

Chapter 36: NATIONALISM AND POLITICAL IDENTITIES IN ASIA, AFRICA, AND LATIN AMERICA

View Set

Major concepts for Unit 5 AP HuG

View Set